Question

In: Advanced Math

11. Given a row echelon form or the reduced row echelon form of an augmented matrix...

11. Given a row echelon form or the reduced row echelon form of an augmented matrix of a system of equations, determine the number of solutions the system has.

Solutions

Expert Solution

Please like it. My CF score is very low. Please....


Related Solutions

The following matrix is in reduced row echelon form. Decode from the matrix the solution of...
The following matrix is in reduced row echelon form. Decode from the matrix the solution of the corresponding system of linear equations or state that the system is inconsistent. (If the system is dependent assign the free variable the parameter t. If the system is inconsistent, enter INCONSISTENT.) 1 0 5 −4 0 1 −8 10 0 0 0 0 (x1, x2, x3) =
Is the following matrix in its reduced row echelon form? Explain your judgement.
Is the following matrix in its reduced row echelon form? Explain your judgement. 
1. For both systems below form the augmented matrix, reduce the augmented matrix to echelon form...
1. For both systems below form the augmented matrix, reduce the augmented matrix to echelon form (zeros below the diagonal), and give the solution to the system. (a) x – 2y + z = 0; 2x + 3y – z = 16; 3x – y – 3z = 23. (b) 2x – 3y + z = -3; x + 2y – 5z = -29; 3x – y + 2z = -2
for matrices, what is the difference between row reduced echelon form and an upper triangular matrix?
for matrices, what is the difference between row reduced echelon form and an upper triangular matrix?
Give an example of an augmented matrix in echelon form corresponding to a system of 2...
Give an example of an augmented matrix in echelon form corresponding to a system of 2 equations in three unknowns satisfying each of the following conditions or explain why it is not possible. (a) No solutions (b) One unique solution (c) Infinitely many solutions.
Find the reduced row echelon form of the following matrices. Interpret your result by giving the...
Find the reduced row echelon form of the following matrices. Interpret your result by giving the solutions of the systems whose augmented matrix is the one given. [ 0 0 3 -1 5 1 0 0 4 2 4 1 3 0 -8 1 2 7 9 0 ]
Write down an augmented matrix in reduced form corresponding to a system with 3 equations and...
Write down an augmented matrix in reduced form corresponding to a system with 3 equations and 5 variables which has infinitely many solutions and 2 free variables. Write down an augmented matrix in reduced form corresponding to a system with 4 equations and 5 variables which has no solutions and 2 free variables.
Given the matrix with rows [1,1,k 1] [1,k,1 1] [k,1,1 -2] Find the reduced row echelon...
Given the matrix with rows [1,1,k 1] [1,k,1 1] [k,1,1 -2] Find the reduced row echelon form of M, and explain how it depends on k. (b) Consider the linear system Ax = b for which the augmented matrix is A b = M. i. For what values of k is the system inconsistent? ii. For what values of k does the system have a unique solution?
Write the following matrices into row echelon form.
Exercise1. Write the following matrices into row echelon form.
Rank of a Matrix Using the Echelon Form.
Find the Rank of a Matrix Using the Echelon Form of the above matrix.Give details step by step.
ADVERTISEMENT
ADVERTISEMENT
ADVERTISEMENT